Answer to Question #192827 in Analytic Geometry for Simphiwe Dlamini

Question #192827

Let L1 be the line in R3 with equation (x,y,z)=(1,0,2) + t(−1,3,4); t∈R and let L2 be the line that is parallel to L1 and contains the point (1,−1,3). Let V be the plane that contains both the lines L1 and L2.

(a)Find two vectors that are both parallel to the plane V but are not parallel to one another.

(b) FindavectorthatisperpendiculartotheplaneV(c) Find an equation for the plane V.(d) Find an equation for the line L3 that is perpendicular to the plane V and contains the point (1,−1,4).


1
Expert's answer
2021-05-13T18:06:53-0400

(a) Let us compose the parametric equations of the lines  L1 and L2 :

"{L_1}:\\left\\{ {\\begin{matrix}\n{x = 1 - t}\\\\\n{y = 3t}\\\\\n{z = 2 + 4t}\n\\end{matrix}} \\right." , "{L_2}:\\left\\{ {\\begin{matrix}\n{x = 1 - t}\\\\\n{y = -1+3t}\\\\\n{z = 3+ 4t}\n\\end{matrix}} \\right."

The direction vector of these lines will be the vector "\\overrightarrow p = \\left( { - 1;3;4} \\right)"

Since the lines are parallel to the plane V, the vector "\\overrightarrow p" is also parallel to the plane.

We also know that

"{M_1}\\left( {1;0;2} \\right) \\in {L_1}" , "{M_2}\\left( {1; - 1;3} \\right) \\in {L_2}"

Since the lines are parallel to the plane V, the vector "\\overrightarrow {{M_1}{M_2}} = \\left( {1 - 1; - 1 - 0;3 - 2} \\right) = \\left( {0; - 1;1} \\right)" is also parallel to the plane.

Since the coordinates of the vectors "\\overrightarrow p" and "\\overrightarrow {{M_1}{M_2}}" are disproportionate, they are not collinear.

Answer: "\\overrightarrow p = \\left( { - 1;3;4} \\right)" , "\\overrightarrow {{M_1}{M_2}} =\\left( {0; - 1;1} \\right)"

(b) The vector perpendicular to the plane is found as the vector product of vectors "\\overrightarrow p" and "\\overrightarrow {{M_1}{M_2}}" :

"\\overrightarrow n = \\left| {\\begin{matrix}\n{\\overrightarrow i }&{\\overrightarrow j }&{\\overrightarrow k }\\\\\n{ - 1}&3&4\\\\\n0&{ - 1}&1\n\\end{matrix}} \\right| = \\overrightarrow i \\left( {3 + 4} \\right) + \\overrightarrow j \\left( {0 + 1} \\right) + \\overrightarrow k \\left( {1 - 0} \\right) = \\left( {7;1;1} \\right)"

Answer: "\\overrightarrow n = \\left( {7;1;1} \\right)"

(с) Since V contains L2, "{M_2}\\left( {1; - 1;3} \\right) \\in V"

Let us make an equation of V for a point "{M_2}\\left( {1; - 1;3} \\right)" and a normal vector "\\overrightarrow n = \\left( {7;1;1} \\right)" :

"7(x - 1) + 1(y + 1) + 1(z - 3) = 0 \\Rightarrow 7x + y + z - 9 = 0"

Answer: "7x + y + z - 9 = 0"

(d) If  L3 is perpendicular to the plane V then L3 is parallel to "n = \\left( {7;1;1} \\right)" .

Then

"{L_3}:\\frac{{x - 1}}{7} = \\frac{{y + 1}}{1} = \\frac{{z - 4}}{1}"

Answer: "{L_3}:\\frac{{x - 1}}{7} = \\frac{{y + 1}}{1} = \\frac{{z - 4}}{1}"


Need a fast expert's response?

Submit order

and get a quick answer at the best price

for any assignment or question with DETAILED EXPLANATIONS!

Comments

No comments. Be the first!

Leave a comment

LATEST TUTORIALS
New on Blog
APPROVED BY CLIENTS